If an activity significantly reduces chronic lower back pain, doctors should be prepared to discuss the merits of tha...

mluna on October 31, 2019

Assumption

Hi, I always have trouble with assumption questions. It's considered a must be true or strengthen question? What's the best way to attack these questions?

Replies
Create a free account to read and take part in forum discussions.

Already have an account? log in

SamA on October 31, 2019

Hello @mluna,

Necessary assumption questions have their own category, separate from must be true and strengthen questions. I'll give you some tips for how to approach them, and then I recommend you watch the video lesson.

The question stem will usually appear as follows:
"Which one of the following is an assumption required by the argument?"
Or
"The conclusion can be properly drawn if which of the following is assumed?"

If you couldn't tell just from reading the argument, this question stem indicates that the argument is not complete. As it stands, the argument cannot be properly drawn. Maybe it is missing important information, or maybe there is a hole in the argument. The correct answer choice, the necessary assumption, will provide this important information or fix the hole in the argument. It is not the same answer type as a strengthen question. Rather than a statement that supports the conclusion, the conclusion depends entirely on the necessary assumption. Without it, the argument fails. Let's look at your question.

Premise 1: If an activity significantly reduces back pain, doctors should be prepared to discuss the merits of that activity with patients.

Premise 2: A recent study showed that yoga and stretching classes lead to equal reductions in lower back pain.

Conclusion: Doctors treating patients with lower back pain should be prepared to discuss the merits of yoga.

What is the hole in this argument? We do not know if yoga significantly reduces lower back pain. We only know that it is equal to stretching classes. They could both have zero effect on lower back pain. Our assumption needs to fix this.

B is the correct answer, because it tells us that yoga does significantly reduce chronic back pain. This means yoga qualifies for premise 1. The doctor should be prepared to discuss it. Our conclusion is now valid. If we do not assume B, the conclusion is not valid.

bb042745 on October 4, 2020

Does the negation technique work on this question / answer choice? if so, can you please explain how. I do not find that intuitively obvious. Thank you.